5
$\begingroup$

What is the limit of the continued fraction whose partial denominators are the composites?

  • 0
    The posting says "below", but it isn't there. Let's see how it looks here: $1+\cfrac{1}{4+\cfrac{1}{6+\cfrac{1}{8+\cfrac{1}{9+ \cfrac{1}{10+\cfrac{1}{12+\cfrac{1}{14+\cfrac{1}{15+\cdots}}}}}}}}$2011-11-18
  • 0
    [This question](http://math.stackexchange.com/questions/63865) is the dual of this question.2011-11-18
  • 0
    Maybe I should have omitted "$1+{}$" at the beginning.2011-11-18

2 Answers 2